You are on page 1of 15

Decision Sciences II

End-Term Examination Solution


Thursday, December 31st, 2020

Total No. of Pages: ? Name: _________________________


Total No. of Questions: 5 Roll #: _________________________
Time: 210 minutes Section: ________________________

Instructions
1. This is an open book, open notes exam.
2. The exam is to be taken individually without any advice or assistance from anyone else.
3. Answer all questions only in the space provided following the question.
4. Show all work and give adequate explanations to get full credit.
5. No clarifications will be made during the exam.
6. Define all the decision variables precisely and label all the constraints clearly.

Question Number Q1 Q2 Q3 Q4 Q5 Total

Max Marks 10 15 20 15 15 75

Marks Scored

I pledge that I have neither given nor received any help during the course of this final examination.
I understand that I need to work on this exam individually and cannot accept help from anyone, in-
person or electronically, and violating these instructions will be penalized as per the IIM-B Student
Honour Code.
Question 1 (10 marks)

Sholay Gaadiwale, Inc., a large automobile manufacturing company, manufactures three different
families of vehicles: family of trucks; family of compact sedans; and family of midsize/luxury
coupes and sedans. One assembly plant outside Ramgarh, KA, assembles two models from the
family of midsize/luxury coupes and sedans.

The “Jay” model is a two-door luxury coupe with leather seats, wooden interiors, customized
features, and navigational capabilities. It is marketed for high-net-worth individuals and generates
a healthy $5400/car profit for the company. The second model, named “Veeru”, is a four-door
entry-level sedan with vinyl seats, plastic interiors, and standard features. It is targeted towards
mid-career professionals with families and generates a modest $3600/car profit for the company.

Basanti Ghodewali, the Vice President of Sholay, Inc., is currently deciding the production schedule
for the next month. The various components required for assembly of the two models are shipped
from various manufacturing plants across the country. She estimates that the assembly plant has a
capacity of 48000 labor-hours for the duration of the month, and it is also known that one Jay car
requires 12 labor-hours of assembly time and one Veeru car requires 6 labor-hours of assembly
time.

Thakur, the forecasting expert, has shared with Basanti that they will only be able to procure 20000
doors (10000 right-hand side doors and 10000 left-hand side doors) from their car door
manufacturing plant. Both Jay and Veeru cars use the same doors. Moreover, Thakur has also
forecasted that the demand for the Jay model will be at most 3000 cars for next month, whereas the
demand for the Veeru model is only limited by the number of cars assembled.

Basanti wishes to run through some scenarios before making any decisions. Towards this end,
answer the following questions:

(a) Formulate a linear programming problem to determine the optimal number of Jay and Veeru
models that need to be assembled at the plant in order to maximize profit. (3 marks)

Let,
𝑥1 = # 𝑜𝑓 𝐽𝑎𝑦 𝑚𝑜𝑑𝑒𝑙𝑠 𝑎𝑠𝑠𝑒𝑚𝑏𝑙𝑒𝑑
𝑥2 = # 𝑜𝑓 𝑉𝑒𝑒𝑟𝑢 𝑚𝑜𝑑𝑒𝑙𝑠 𝑎𝑠𝑠𝑒𝑚𝑏𝑙𝑒𝑑

LP: Maximize 5400𝑥1 + 3600𝑥2


subject to: 12𝑥1 + 6𝑥2 ≤ 48000 (assembly)
𝑥1 + 2𝑥2 ≤ 10000 (doors)
𝑥1 ≤ 3000 (demand limit for Jay)
(𝑥1 , 𝑥2 ) ≥ 0 (non-negativity)
(b) The graphical representation of the linear program formulated in part (a) is given below. Identify
the coordinates of all the extreme points (labeled 1-5) and determine the optimal product mix
and the optimum objective function value. (2 marks)

Coordinates of the extreme points are given by:

#1: (2000, 4000); #2: (3000, 2000); #3: (0, 5000); #4: (3000, 0); #5: (0, 0).

The optimum solution occurs at extreme point #1: (2000, 4000), i.e., 2000 Jay cars, 4000
Veeru cars, with 𝑧 ∗ = $2,52,00,000.

(c) Ram Sippy, the Chairman of the Board of Directors for Sholay, Inc., wishes to capture a larger
share of the luxury car market, and therefore wishes to meet the full demand for the Jay model.
By how much would the profit of the assembly plant reduce as compared to part (b) if they
pursue this strategy? Ram Sippy has indicated that the company will take on this strategy if the
profit reduction is lesser than $1,500,000. Should the company adopt this strategy?
(2 marks)

To meet the full demand of the Jay model, we set 𝑥1 = 3000, and the company must now
operate at the product mix of (3000, 2000) cars, i.e., optimal solution in this case switches to
extreme point #2. therefore, profit will drop to $2,34,00,000. Reduction in profit is $1,800,000,
which is more than what Ram Sippy bargained for; so, company will not adopt this strategy.
(d) Gabbar, the assembly plant foreman knows that as per the latest round of negotiations with the
labor union, management can demand an increase in the labor-hour capacity as and when
required. How much additional profit can be made as compared to part (b) for a unit increase in
labor-hours? What is the maximum increase in labor-hours for which this increase in profit is
valid? (3 marks)

Using dual formulation (or MS Excel Sensitivity report), we get the shadow price = 400 for the
assembly constraint. Therefore, a one-unit increase in labor-hours leads to a $400 increase in
profit, and this increase is valid for an additional 9000 hours.
Question 2 (15 marks)

The Wonder Sports Supply Company produces All-Pro (A) and College (C) footballs at its
manufacturing plant in Jalandhar. The maximum demand from wholesalers for the months of
February and March is as follows:

Jalandhar
Product
February March
A 3600 6300
C 4500 5400

The plant has 20 and 23 production days available in February and March, respectively, and an
inventory capacity of 1000 footballs (for A and C models combined) is available in any given
month. The inventory holding cost is Rs. 30/- per unit for A and Rs. 40/- per unit for C.
There are two production processes that can be used at the plant to produce the footballs at the
following production costs (Rs. per unit):

Plant 1
Product
Process 1 Process 2
A 620 590
C 780 850

The number of footballs that can be produced per day varies according to the process and are as
given below:
Plant 1
Product
Process 1 Process 2
A 100 140
C 120 150

The net sales revenue (selling price – shipping costs) are Rs. 830/- per unit for A and Rs. 1120/-
per unit for C. Wonder Sports would like to determine the optimal production and sales plan that
will maximize its total profit.
(a) Does Wonder Sports have enough capacity to meet all the demand? (1 point)

The capacity is not sufficient to meet the demand.

Max production possible in Feb and March (only C is produced using Process 2 which
gives the highest output): 3000, 3450 (days*production rate/day)
(b) Define the decision variables required to determine the optimal production and sales plan for
Wonder Sports. (3 points)

Decision Variables:
Quantity Produced (Product/Process) (Product A - 1, C - 2)

February March
F F M M
X11 X12 X11 X12
F F M M
X21 X22 X21 X22

Quantity Sold of Product 1/2 in Feb/March respectively: Sx1F, Sx2F, Sx1M, Sx2M
Quantity held in Inventory of Product 1/2 respectively in Feb: Ix1F, Ix2F

(c) State the constraints and label them clearly. (8 points)

Capacity:
XF
11 XF XF XF
+ 12 + 21
+ 22
≤ 20 (Days available in sFeb)
100 140 120 150

XM
11 XM
12 XM
21 XM
22
+ + + ≤ 23 (Days available in March)
100 140 120 150

Inventory Balance:
F F
X11 + X12 = Sx1F + Ix1F (Product 1 Feb)
M M
X11 + X12 + Ix1F = Sx1M (Product 1 March)
F F
X21 + X22 = Sx2F + Ix2F (Product 2 Feb)
M M
X21 + X22 + Ix2F = Sx2M (Product 2 March)

Demand:
Sx1F ≤ 3600, Sx1M ≤ 6300; Sx2F ≤ 4500, Sx2M ≤ 5400

All vars ≥ 0 (non-neg)

(d) State the objective function. (3 points)


Max Profit = Sales Revenue – Production Cost – Inventory Cost

[830(Sx1F + Sx1M ) + 1120(Sx2F + Sx2M )] - [620(X11


F M
+ X11 F
)+ 590(X12 +
M F M F M F F
X12 ) + 780(X 21 + X21 + 850(X22 + X22 )] - [30∗ Ix1 + 40 ∗ Ix2 ]
Question 3 (20 marks)

Ranchoddas Shamaldas Chanchad (Rancho) established CNG (Cleanliness is Next to Godliness), a


manufacturing firm that specialized in manufacturing hand sanitizers in 2020. Sanitizers
manufactured by CNG are made of three main ingredients: (i) Benzalkonium Chloride (BC); (ii)
Ethyl alcohol (EA); and (iii) Isopropyl alcohol (IS). CNG makes three different varieties of hand
sanitizers namely: Regular, Premium, and OCD. The ingredients used (in ml) to produce one litre
of sanitizer is shown in the following table, along with demand for different sanitizers and
availability of ingredients.

Ingredient in ml for 1 litre of Sanitizer Minimum Profit from


Sanitizer Demand for Sanitizer
BC EA IS
Sanitizer (in litres) (Rs/litre)
Regular A11 600 200 75 40
Premium 100 300 450 80 60
OCD 100 400 A33 20 100
Availability of
20000 80000 100000
ingredients (in ml)

The corresponding LP which maximizes profit subject to resource availability and demand
constraints is formulated below:

Maximize 40𝑥1 + 60𝑥2 + 100𝑥3


subject to:
𝐴11 ∗ 𝑥1 + 100 𝑥2 + 100𝑥3 ≤ 20000 (constraint for BC)
600𝑥1 + 300𝑥2 + 400𝑥3 ≤ 80000 (constraint for EA)
200𝑥1 + 450𝑥2 + 𝐴33 ∗ 𝑥3 ≤ 100000 (constraint for IS)
𝑥1 ≥ 75 (constraint for Regular Sanitizer)
𝑥2 ≥ 80 (constraint for Premium Sanitizer)
𝑥3 ≥ 20 (constraint for OCD Sanitizer)
𝑥𝑖 ≥ 0 𝑓𝑜𝑟 𝑖 = 1, 2, 𝑎𝑛𝑑 3.

The Excel Solver sensitivity output is given below:


Variable Cells
Final Reduced Objective Allowable Allowable
Cell Name Value Cost Coefficient Increase Decrease
$A$2 X1 75 0 40 110 1E+30
$B$2 X2 80 0 60 15 1E+30
$C$2 X3 27.5 0 100 1E+30 20

Constraints
Final Shadow Constraint Allowable Allowable
Cell Name Value Price R.H. Side Increase Decrease
$A$4 BC 14500 20000 1E+30 5500
$A$5 EA 80000 0.25 80000 22000 3000
$A$6 IS 64750 100000 1E+30 35250
$A$7 Regular 75 75 5 55
$A$8 Premium 80 80 10 80
$A$9 OCD 27.5 20 7.5 1E+30

(a) Determine the values of A11, A22 and A33 in the LP formulation. (3 points)

From the output, the final values of the corresponding constraints are: 14500, 80000 and 64750
respectively. That is:

A11 * X1 + 100 X2 + 100X3 = 14500 (constraint for BC)

600X1 + 300* X2 + 400X3 = 80000 (constraint for EA)

200X1 + 450X2 + A33* X3 = 64750 (constraint for IS)

Solving each equation, we will get:

A11 = 50, A22 = 300 and A33 = 500.

Use the primal-dual relationship and complementary slackness theorem to answer questions 2 to 8
(b) What is the shadow price for the Benzalkonium Chloride constraint? (1 point)

0 since the constraint is non-binding

(c) Write the dual to the LP primal problem formulation provided above. Using the
complementary slackness theorem, calculate the shadow price values of constraints, Regular
and Premium. (6 points)
Dual of the Primal LP is:
Min 20000y1 + 80000y2 + 100000y3 – 75y4 – 80Y5 – 20Y6
50Y1 + 600Y2 + 200Y3 - Y4 >= 40

100Y1 + 300Y2 + 450Y3 – Y5 >= 60


100Y1 + 400Y2 + 500Y3 – Y6 >= 100

All three constraints are binding (all decision variables are basic) and values of Y1 = Y3 = Y5 =
0. The equations can be written as:
600 * Y2 - Y4 = 40

300 *Y2 – Y5 = 60
400Y2 = 100
Solving we get: Y4 = 110 (or – 110 non standard form) and Y5 = 15 (or -15 in non-standard
form).

(d) Due to Covid-19, the profit earned from regular sanitizer has increased from 40 to 100, what
will be the impact of this change on the optimal solution and the profit? (2 points)

Allowable increase is 110, the acutal change (60) is within the range. So the optimal solution
will remain same and the profit will increase by 75*60.

(e) CNG was informed that one of the suppliers of Ethyl Alcohol can supply additional 20000
ml of EA. Should CNG buy this additional quantity of EA? What will be the impact of this
additional quantity on the profit? (2 points)

Allowable increase for RHS of EA constraint is 22000. So, the profit will increase by 20000*0.25
= 5000.

(f) Due to increased demand for sanitizers, CNG has increased the price such that the profit of
regular is 60, premium is 90 and OCD is 150. What will be the impact of this increase on
the profit? (2 points)

The profits are increased by 50% across all products, thus there is no change in the slope of
the objective function and this there will be no change in the optimal solution.

(g) There is a demand for 5000ml BC in the open market at INR 2 per ml. Should CNG sell
BC directly in the market? (1 point)
Slack for BC constraint is 5500, so they should sell 5000ml of BC in the open market.

(h) CNG would like to introduce a new Luxury Sanitizer, which requires 200ml of BC, 500ml
of EA and 400ml of IS. The profit from this new Sanitizer is 140. Should CNG introduce
the new Sanitizer? (3 points)

The new constraint is:

200Y1 + 500Y2 + 400Y3 >= 140


The constraint is infeasible, thus the current optimal solution is not optimal
Question 4 (15 marks)

The Board of Directors for Cobra, Inc. is currently evaluating seven large potential business
ventures. The amount of capital investment required to participate in these business ventures as
well as the profit they yield after three years (in millions of dollars) is specified in Table 1 below.

Table 1. Required capital and profit yield for seven business ventures
Business Venture
1 2 3 4 5 6 7
Capital Required 43 28 34 48 17 32 23
Estimated Profit 17 10 15 19 7 13 9

Cobra Inc. has a total amount of $100 million of capital available for investment into these new
business ventures. Using this information, answer the following questions:

(a) Formulate an integer program to determine which of these business ventures should be selected
by Cobra Inc. to maximize their total profit at the end of three years subject to their budget
constraint. Clearly define your decision variables, and then write the constraints and the
objective function. (3 marks)

Let,
1, 𝑖𝑓 𝐶𝑜𝑏𝑟𝑎 𝑖𝑛𝑣𝑒𝑠𝑡𝑠 𝑖𝑛 𝑣𝑒𝑛𝑡𝑢𝑟𝑒 𝑖
𝑥𝑖 = { .
0, 𝑜𝑡ℎ𝑒𝑟𝑤𝑖𝑠𝑒

IP: Maximize 17𝑥1 + 10𝑥2 + 15𝑥3 + 19𝑥4 + 7𝑥5 + 13𝑥6 + 9𝑥7


subject to: 43𝑥1 + 28𝑥2 + 34𝑥3 + 48𝑥4 + 17𝑥5 + 32𝑥6 + 23𝑥7 ≤ 100
𝑥𝑖 ∈ {0, 1}, 𝑖 = 1, … , 7 .

(b) Using MS Excel Solver, determine the optimal solution to the problem. Upload your MS Excel
file showing the formulation and calculations. (3 marks)

The optimal solution to the problem is 𝑥3 = 𝑥4 = 𝑥5 = 1, with 𝑧 ∗ = 41.

(c) If Cobra Inc. invests in Venture 3, then they must invest in Venture 5. What constraint needs
to be added to the formulation in part (a) to reflect this condition? Will this constraint change
your optimal solution from part (b)? (2 marks)

Constraint: 𝑥5 ≥ 𝑥3 .

This will not affect the optimal solution as 𝑥3 and 𝑥5 are already included in the current optimal
solution.
(d) Suppose that Cobra Inc. can invest either in Venture 3 or Venture 4 but not both. What
constraint needs to be added to the formulation in part (a) to reflect this condition? Will this
condition change the optimal solution from part (b)? (2 marks)

Constraint: 𝑥3 + 𝑥4 ≤ 1 .

Note that in the current optimal solution, 𝑥3 = 𝑥4 = 1, and this solution violates the above
constraint. Therefore, imposing the above constraint will lead to a new optimal solution.

(e) If Cobra Inc. invests in at least two of Ventures 1, 2, 3, or 4 then they must invest in at least
one of Ventures 5, 6, or 7. What constraint needs to be added to the formulation in part (a) to
reflect this condition? (2 marks)

Condition: If 𝑥1 + 𝑥2 + 𝑥3 + 𝑥4 ≥ 2, then 𝑥5 + 𝑥6 + 𝑥7 ≥ 1.

Solution 1:
1
𝑥5 + 𝑥6 + 𝑥7 ≥ (𝑥1 + 𝑥2 + 𝑥3 + 𝑥4 − 1)
3

Solution 2:
3𝛿 ≥ 𝑥1 + 𝑥2 + 𝑥3 + 𝑥4 − 1
𝛿 ≤ 𝑥5 + 𝑥6 + 𝑥7

Solution 3:
𝑥5 + 𝑥6 + 𝑥7 ≥ 𝑥1 + 𝑥2 + 𝑥3 − 1
𝑥5 + 𝑥6 + 𝑥7 ≥ 𝑥2 + 𝑥3 + 𝑥4 − 1
𝑥5 + 𝑥6 + 𝑥7 ≥ 𝑥1 + 𝑥3 + 𝑥4 − 1

(f) If Cobra Inc. invests in at least two of Ventures 1, 2, or 3, then they must invest in at most two
of Ventures 4, 5, 6, or 7. What constraint needs to be added to the formulation in part (a) to
reflect this condition? (3 marks)

Condition: If 𝑥1 + 𝑥2 + 𝑥3 ≥ 2, then 𝑥4 + 𝑥5 + 𝑥6 + 𝑥7 ≤ 2.

Solution 1:
𝑥1 + 𝑥2 + 𝑥3 + 𝑥4 + 𝑥5 + 𝑥6 ≤ 4
𝑥1 + 𝑥2 + 𝑥3 + 𝑥4 + 𝑥6 + 𝑥7 ≤ 4
𝑥1 + 𝑥2 + 𝑥3 + 𝑥5 + 𝑥6 + 𝑥7 ≤ 4

Solution 2:
2𝛿 ≥ 𝑥1 + 𝑥2 + 𝑥3 − 1
𝑥4 + 𝑥5 + 𝑥6 + 𝑥7 ≤ 2 + 2(1 − 𝛿)
Question 5 (15 marks)

Consider the following integer programming problem:

Minimize −𝑥1 − 𝑥3 + 𝑥4
subject to:
−2𝑥1 + 7𝑥2 + 3𝑥3 + 𝑥4 ≤ 6
7𝑥1 + 3𝑥2 − 4𝑥3 − 2𝑥4 ≤ 1
2𝑥2 + 3𝑥3 + 6𝑥4 ≥ 5
−3𝑥1 + 2𝑥3 ≤1
𝑥𝑖 ≥ 0 𝑎𝑛𝑑 integer, 𝑖 = 1, 2, 3, 4.

You are given a partially completed branch-and-bound tree below, where the labels at the center of
the nodes indicate the order in which the nodes were evaluated, i.e., its LP relaxation was solved.
The notation 𝑧 𝑘 = 𝑞 indicates that a linear programming relaxation at the node labeled k was solved
and the objective function value is q. The notation 𝑥 𝑘 =   (𝑎,  𝑏,  𝑐,  𝑑) indicates that the solution
to the LP relaxation at node k is given by 𝑥1 =  𝑎, 𝑥2 =  𝑏, 𝑥3 =  𝑐, 𝑥4 =  𝑑. An active node is
defined as one that still needs to be explored and cannot be fathomed. At any stage, the incumbent
solution is the best-known integer (feasible) solution to the problem.

You are required to complete the branch-and-bound tree and determine the optimal solution to the
problem. Towards this end, answer the following questions:

(a) What is the current incumbent solution? (1 point)

Current incumbent solution is at Node 2: 𝑥 2 =   (3,  0,  2,  6) with 𝑧 2 = 1.

(b) What is the best lower bound on the problem at this stage? (1 point)

Best lower bound on the problem is −4.857 at Node 3.

(c) What are the active nodes in the branch-and-bound tree at this stage? (1 point)

Active nodes are Nodes 5, 7, 8


(d) Formulate and use MS Excel Solver to solve the linear programming relaxation at Node 5.
Upload your MS Excel file showing the formulation and calculations. (4 points)

Adding the constraints 𝑥1 ≤ 1 and 𝑥3 ≤ 3 to the original problem, we get the LP relaxation
at Node 5. Solving this, we get: 𝑥 5 =   (1,  0,  2,  0) with 𝑧 5 = − 3 .

(e) Based on your solution to part (d), did the incumbent value change? If yes, what is the new
incumbent solution? (1 point)

The incumbent solution gets updated to 𝑥 5 =   (1,  0,  2,  0) with 𝑧 5 = − 3 .

(f) What are the active nodes in the branch-and-bound tree at this stage? (1 point)

Active nodes in the tree are Node 7 and Node 8.

(g) From the solution shown at Node 6, select an appropriate branching variable and determine the
optimal solution at Node 8. Did your incumbent solution change? (4 points)

The branching variable selected at Node 6 is 𝑥4 . Setting 𝑥4 ≥ 1, we get to Node 8, and the
LP solution at this node yields: 𝑥 8 =   (2,  0,  3,  1) with 𝑧 8 = − 4.

(h) Can the branch-and-bound process be terminated at this stage? If yes, explain your reasoning.
If not, continue branching and determine the optimal solution to the problem. (2 points)

Yes, the branch-and-bound process can be stopped at this stage. The only other active node at
this stage is Node 7. But, Node 6 has a lower bound of 𝑧 6 = − 4.5 , and therefore the best
integer solution that we can possibly be obtained by solving Node 7 cannot be better than the
current incumbent. Therefore, Node 7 can be fathomed by bound and we can declare that 𝑥 8 =
 (2,  0,  3,  1) is the optimal solution with 𝑧 8 = − 4 as the optimum objective function value.

You might also like